(7−2)3×(24−11)? cause im in 6th grade lol

Answers

Answer 1

Answer:

(7−2)(3)(24−11)

=(5)(3)(24−11)

=15(24−11)

=(15)(13)

=195

Step-by-step explanation:


Related Questions

Mateus recorded the number of miles driven and the amount
of gasoline used in the table below.
Gas Mileage
Distance Gas Usage
(in miles) (in gallons)
105
5
210
10
315
15
420
20
At this rate, what is the maximum miles that can be driven
with 12 gallons of gasoline?
A) 220 miles
B) 236 miles
C) 252 miles
D) 262.5 miles

Answers

Answer:

C. 252 miles.

Step-by-step explanation:

Since we already know how many miles is driven for 10 gallons of gas, we only need to find 2.

Since 105 miles runs on 5 gallons, divide 105 by 5 to find out how many miles runs on 1 gallon.

105/5=21

Then, multiply that by 2 to find out how many miles run on 2 gallons of gas.

21x2=42.

Finally, add 42 to 210 to get the amount of miles that runs on 12 gallons of gas.

210+42=252

Sarah has only $180 to spend on her birthday party. If she had to pay $100 for the room and each guest cost $6
for food, how many people can she invite without going over her budget?

Answers

Answer:

She can invite 13 people.

Step-by-step explanation:

Because Sarah has to pay $100 for the room, we can subtract.

$180-$100=$80.

Now that she has $80 left, we can see how many times $6 can go into $80.

$6x14=84.

This number is four more than $80, so Sarah can not invite 14 people.

But, if we do $6x13, we get $78.

$78 does not past $80 and is really close, Sarah can invite 13 people.

Final Answer: Sarah can invite 13 people without going over her budget.

I hope this helps and have a wonderful day! ;0

Answer:

13

Step-by-step explanation:

180-100 = 80

80 divided by 6 = 13.333

What is the value of x in the polygon below?

Answers

Answer:

x=141

Step-by-step explanation:

The measure of all the angles in a septagon (7 sided shape) add up to 900°. The current sum of the known angles 131°+122°+125°+148°+107°+126°=759°.

900° total sum minus known sum 759° equals 141°.

Given the points (4, 5) and (-2, 8), find the slope of the line that passes through the points.

Group of answer choices

9/6

-2/1

-1/2

1/2

Answers

Answer: c

Step-by-step explanation:

Formula of slope of line :

m=y2-y1/x2-x1

m=8-5/-2-4

=-3/6

m=-1/2

Select the correct answer.

Which expression in factored form is equivalent to this expression?

Answers

what is the expression?

The ages and grades of some of the 19 girls on a club soccer team are shown in the table. Which two-way frequency table correctly shows the marginal frequencies? ​

Answers

Answer:

Step-by-step explanation:

add up to a total of 21 girls. The question says there are 19 girls, so those two are eliminated. Table B's data doesn't even add properly so that is eliminated too

Answer:

Add up the 21 girls,

table B is out btw

Ik I'm late just want to help :)

Which of the following points lie in the solution set to the following system of inequalities? y > â’3x 3 y > x 2 (2, â’5) (â’2, 5) (2, 5) (â’2, â’5).

Answers

Answer:

The concept of identity, the meaning we give to ourselves and our sense of purpose, connects us to others. This concept best illustrates the link between the psychological and ____ system

A number minus 33 is at least -5

Answers

Answer:

28

Step-by-step explanation:

Put in calculator 28-33= -5

Erin bought 4 jars of jelly and 6 jars of peanut butter for $19.32. Adam bought 3 jars of jelly and 5 jars of peanut butter for $15.67. Find the cost of a jar of peanut butter?

Answers

Answer:

the jar of peanut butter is $2.36

Step-by-step explanation:

Answer: $2.36

Step-by-step explanation:

A set of 6 art books costs $241.25. A copy of one of the books, bought separately, costs $38.25. About how much less is the cost of the 6 books if you buy the set?

Answers

The cost of the 6 books if you buy the set is $11.75 less.

The given parameters:

Total number of books, n = 6Cost of the books, c = $241.25Cost of one book, = $38.25

The total cost of the 6 books when bought separately is calculated as follows;

Cost = 6 x $38.25

Cost = $229.5

The difference in price is calculated as follows;

= $241.25 - $229.5

= $11.75

Thus, the cost of the 6 books if you buy the set is $11.75 less.

Learn more about cost price here: https://brainly.com/question/19104371

Walmart has white tees and joggers on sale. The white tees cost $4 each and the joggers cost $6
each. You have $36 to spend. The equation 4x + 6y = 36 can model this situation where x
represents the number of white tees and y represents the number of joggers purchased.
Graph the equation and identify (3) possible combinations of white tees and joggers that can be
bought.
cost = price x quantity
4 × X
Combination

Answers

y intercept = 6
Slope = -2/3

6 joggers and 0 tees
4 joggers and 3 tees
2 joggers and 6 tees
0 joggers and 9 tees

Dan is using tiles to add - 8 +6. He begins with the tiles shown below.
OOOOOOOO
What is the sum of -8+6?
0-14
-2
02
14

Answers

Answer:

-2

Step-by-step explanation:

help me with this plss

Answers

Answer:

31-13=18

Step-by-step explanation:

so its 18 pls give brainliest

Answer:

18

Step-by-step explanation:

Since Robyn bought 13 video games and got 31 at the end, if you take away 13 then you will get 18 meaning the answer is 18

Maths Sum:

There's 2 ways you could go this, there's the algebraic way of doing it (that I don't recommend for this type of question and is a bit to advanced right for this question also) which is:

x + 13 = 31

  - 13   -13

x = 18

And there's the simple way of doing that I recmmend for this type of question which is:

31 - 13 = 18

Find a counterexample for this statement.

A quadrilateral has four congruent sides.

Answers

Answer:

A Rhombus

Step-by-step explanation:

it's a quadrilateral, but its sides aren't congruent

pls someone help me.

Answers

AB=EF

ABEF=ABF+AEF

NOW CONTINUES THE SOLUTION

Please please help!!

Answers

Step-by-step explanation:

get add and add on ok

Please help! If possible please show your work. Thank you in advance!! :D

Answers

Answer:

(-4)³ = -64-64

Step-by-step explanation:

The first step in solving the equation is to cube both sides:

  (∛x)³ = (-4)³ . . . . . = (-4)(-4)(-4) = 16(-4) = -64

  x = -64 . . . . . simplified

__

We're not sure what "checking" is supposed to involve here. Usually, one would check the answer by seeing if a true statement is made when the answer is put into the original equation.

  ∛(-64) = -4 . . . true

Many calculators will not compute √(-64) because they compute roots using logarithms. The log of a negative number is not defined.

So, the way one would check this is to cube both sides, which is how we got the answer in the first place. We expect the same result from doing the same operation again, so it isn't really a check.

What is the slope of this line?
A.) −1/2
B.) 1/2
C.) 2
D.) I don't know.

Answers

Answer:

2

Step-by-step explanation:

I have no clue

Question 7

Answers

Answer: 27[tex]a^{-6}b^{3}[/tex]

Step-by-step explanation:

you can isolate the separate terms so you get

3^3 x a^(-2x3) x b^3 = 27 a^-6b^3

Davens pays $50 each week for guitar lessons. How much will he pay after 3/4 of a year

Answers

You’ve got to work out 3/4 of the year first, that’s 75%, after you need to work out how many weeks are in those days so you divide it by 7 then divide that answer by $50 and hopefully you get your answer. hope this helps
he will pay $1950.
there are 52 weeks in a year and 52x.75=39
since it is $50 per lesson you do 39x50=1950, so this is how much he pays after 3/4 of a year.

Wesley Snipes earns a monthly salary of $1,685, plus a 8.5% commission on all sales over $2,000 each month. This month, his sales were $6,250. What was his total
income for the month?
$2,224.50
$2,175.80
$2,112.90
$2,046.25
None of these choices are correct

Answers

Answer:

none of theses choices. wesley makes $2185 this month

Step-by-step explanation:

Which of the following is the measure of an acute angle? O A. 112° O B. 90° O C. 78° O D. 182​

Answers

Answer:

C

Step-by-step explanation:

because acute angles have to be less than 90 degrees so therefore it's C.

Answer:

it would be c 78 degrees because under 90 degrees makes a acute angle anything that is under 90 degrees makes acute and anything over 90 degrees makes obtuse angle so c

Step-by-step explanation:

-10x + y = 4
help i need the steps on how to solve this pls​

Answers

Answer:

slope = -10

Step-by-step explanation:

Rearrange the equation by subtracting what is to the right of the equal sign from both sides of the equation :

-10*x-y-(4)=0

1.1 Pull out like factors :

-10x - y - 4 = -1 • (10x + y + 4)

2.1 Solve -10x-y-4 = 0

Tiger recognizes that we have here an equation of a straight line. Such an equation is usually written y=mx+b ("y=mx+c" in the UK).

"y=mx+b" is the formula of a straight line drawn on Cartesian coordinate system in which "y" is the vertical axis and "x" the horizontal axis.

In this formula :

y tells us how far up the line goes

x tells us how far along

m is the Slope or Gradient i.e. how steep the line is

b is the Y-intercept i.e. where the line crosses the Y-axis

The X and Y intercepts and the Slope are called the line properties. We shall now graph the line -10x-y-4 = 0 and calculate its properties

Notice that when x = 0 the value of y is 4/-1 so this line "cuts" the y axis at y=-4.00000

y-intercept = 4/-1 = -4.00000

When y = 0 the value of x is 2/-5 Our line, therefore "cuts" the x-axis at x=-0.40000

x-intercept = 4/-10 = 2/-5 = -0.40000

Slope is defined as the change in y divided by the change in x. We note that for x=0, the value of y is -4.000 and for x=2.000, the value of y is -24.000. So, for a change of 2.000 in x (The change in x is sometimes referred to as "RUN") we get a change of -24.000 - (-4.000) = -20.000 in y. (The change in y is sometimes referred to as "RISE" and the Slope is m = RISE / RUN)

Slope = -10

i hope this helps

Answer:

-10x+ y=4

-10x+y-4=0

10x-y+4=0

Assume that you are going to open a checking account. You are examining different banks and banking accounts to choose a bank account for your needs. Which of the following statements should not be one of your reasons for examining different banks?

a. Being more informed leads to being better prepared to make financial decisions

b. Bank E offers you a chocolate bar for filling out an application for checking and credit at the same time.

c. Banks could have charges that you do not know about until reading carefully.

d. You will become more aware of how well the banks meet your particular needs.

Answers

Answer:the answer is b

Step-by-step explanation:

i took the test

Which system of equations is graphed on the coordinate plane below?

Answers

Answer: 12,2

Step-by-step explanation:

Answer:

so the answer is TRUE

Step-by-step explanation:

I'm not sure how to explain but ik it it

HELPPPPPPP PLSSSSS!!!!!! 100 points
Given: Angle T S R and Angle Q R S are right angles; Angle T Is-congruent-to Angle Q
Prove: Triangle T S R Is-congruent-to Triangle Q R S

Triangles T S R and Q R S share side S R. Angles T S R and S R Q are right angles. Angles S T R and S Q R are congruent.

Step 1: We know that Angle T S R Is-congruent-to Angle Q R S because all right angles are congruent.
Step 2: We know that Angle T Is-congruent-to Angle Q because it is given.
Step 3: We know that Line segment S R is-congruent-to line segment R S because of the reflexive property.
Step 4: Triangle T S R Is-congruent-to Triangle Q R S because

of the ASA congruence theorem.
of the AAS congruence theorem.
of the third angle theorem.
all right triangles are congruent.

Answers

Answer:

B.of the AAS congruence theorem.

Step-by-step explanation:

hope this helps!

10. Given m and b, write the equation of the line:
m= -4; b = 2

Answers

Answer:

y = -4x + 2

Step-by-step explanation:

huang 3 shirts that each cost the same amount, a pair of pants that cost $12, and he pays with a $100 bill. Which
expression represents the amount of change Huang should receive? Select three options.
100-(3)
100--12)
(100-12) - 3
(100-12)*x*)
100-x) - 12

Answers

Answer:

(100-12) - 3

Step-by-step explanation:

we dont know what the cost of the shirts are so that one should be it.

A pair of jeans is on sale for 20% off. The sale price is $55.00. What is the original amount of the pair of jeans?

Answers

Answer:

$ 275

Step-by-step explanation:

20/100 = 55/x (20% is 55 of what number) (%/100 = is/of)

(100 * 55) / 20 = 275

Answer:

68.75

Step-by-step explanation:

In a class of 50 students, 28 are boys and the rest are girls. The ratio of boys to girls in its simplest form is

Answers

14 boys and 11 girls. The ratio would be 14:11
Other Questions
Which of the following effects does the large polecat-ferret mentioned toward the end of the final paragraph have on the development of Conradins character? How greenhouse gases can lead to the formation of damaging acid rain In six years Xavier will be 3 times older than Joanne is today. Joanne is 24. How old is Xavier? 1. Declare a variable named num with aninitial value of 0. Create a while loop thatwill print "Keep going" as long as the valueof num is less than 5. Use window.alert()for displaying the output.2. Create a function named showProd thatreturns the product of n1 and n2.3. Using the new keyword, create an arraynamed info with three (3) values: your firstname, your nickname, and your last name. SSO software is usually used what places? What does it mean when thefrequency of a wave increases?A. the amplitude increasesB. the amplitude decreasesC the wavelength becomes shorterD. the wavelength becomes longer WILL MARK BRAINLIEST!!! PLS HELP. ANY LINKS OR WRONG ANSWERS WILL BE REPORTED !! Read the writers original draft of this sentence from paragraph 4. 4+(3x6)-7 help please Complete the shape below so that AB is a line of symmetry A heating curve is a graph that shows the temperature change of a substance asthermal energy is introduced.TrueFalse outside of our moon which moon in the solar system is most studied Heart, 5 stars, and Brainiest to first right answer!In an experiment, equal amounts of water and sand were heated under a lamp. The initial and final temperatures of each were recorded. A partial record of the temperature is shown.Experimental RecordSubstance Initial Temperature Final TemperatureSand 20 C 26 CWater 20 C ?Which statement about the final temperature of water is correct?A. It will be less than 26 C as sand has a relatively higher specific heat than water.B. It will be less than 26 C as water has a relatively higher specific heat than sand.C. It will be more than 26 C as sand needs relatively higher energy than water to raise its temperature.D. It will be more than 26 C as water needs relatively higher energy than sand to raise its temperature. Which term refers to a secure systems ability to protect against. What is the slope A) 1/3B) 3C) 2 Hercules differ from the other gods was franklin roosevelt a hero to be admired or a tryant who wrongly overstepped his constitutional authority Please help !!!!!!!!1) Write the equation for the reaction between the hydrocarbon and bromine using fully displayed formulae for the hydrocarbon and the organic product.2) The reaction between methane and bromine gas in the presence of UV light also causes the bromine to lose color.(a) Write an equation for this reaction(b) This reaction is described as a substitution reaction. whereas the reaction between X and bromine is an addition reaction. Using the equations you have already written , explain the difference between addition and substitution You hang a light in front of your house using anelaborate system to keep the 12-kg object in staticequilibrium (Figure 1). What are the magnitudes of theforces that the ropes must exert on the knot connectingthe three ropes if 02 = 639 and 03 = 45 ? what are the regions for nepali citizens going for foreign employement?